subject
Physics, 05.05.2020 08:00 akornegay2

If we assume that M87* is not rotating (although it definitely IS rotating but this assumption gives us a much easier calculation that can be done quickly), what is the size of its event horizon (i. e., its Schwarzschild radius)? The Schwarzschild radius of a black hole is given by LaTeX: r_{s\:}=\frac{\:2GM}{c^2}r s = 2 G M c 2 The mass of the black hole is LaTeX: 6.5\:\times10^{9\:}\:\mathrm{M_{\od ot}}6.5 Γ— 10 9 M βŠ™, where the solar mass is LaTeX: M_{\odot}\:=\:2\:\times10^{30}\:kgM βŠ™ = 2 Γ— 10 30 k g Give your answer in AU. For reference, LaTeX: 1\:AU\:=\:1.5\:\times10^{11}\:m1 A U = 1.5 Γ— 10 11 m and the gravitational constant is LaTeX: G\:=\:6.67\:\times10^{-11}\:\:m^{3\ :}kg^{-1}s^{-2}G = 6.67 Γ— 10 βˆ’ 11 m 3 k g βˆ’ 1 s βˆ’ 2 You should also read over section 2.1 in the lab script. Be careful with your units! Do not use equation 1 in the lab script; that equation uses an older estimate of the black hole's mass.

ansver
Answers: 1

Another question on Physics

question
Physics, 21.06.2019 18:00
Aproton is projected toward a fixed nucleus of charge +ze with velocity vo. initially the two particles are very far apart. when the proton is a distance r from the nucleus its velocity has decreased to 1/2vo. how far from the nucleus will the proton be when its velocity has dropped to 1/4vo?
Answers: 3
question
Physics, 21.06.2019 19:30
Density of salty water as compared to the density of pure water is(a) is less(b) is always equal(c) is always more(d) may be less or more
Answers: 1
question
Physics, 22.06.2019 05:00
Does work output exceed work input when a machine is used? a) yes, it does, as machines create energy. b) it depends on the type of machine used. c) no, it doesn’t, as machines are inefficient. d) no, it doesn’t, as machines cannot create energy.
Answers: 1
question
Physics, 22.06.2019 08:30
A17,250 kg rocket is pushed with a thrust of 6,450,284 n. what is the acceleration of the rocket?
Answers: 1
You know the right answer?
If we assume that M87* is not rotating (although it definitely IS rotating but this assumption gives...
Questions